Ebene der oberen Kletterwand


\(\\\)

Aufgabe 1 Zahl a

Wir gleichen die Ebene \(E_a\) der Ebenengleichung

\( \quad E : \; 4x_1+4x_2 - 3x_3 = -8 \)

\(\\\)

an.

\( \quad \begin{array}{ r r c l l } E_8: & x_1 + x_2 - ax_3 & = 1 - 4a & | \cdot 4 \\[6pt] & 4x_1 + 4x_2 - 4ax_3 & = 4 - 16a \\ \end{array} \)

\(\\\)

Damit \(E_a = E\) ist, muss gelten:

\( \quad \begin{array}{ r r c r } \textrm{I} & 4a & = & 3 \\[6pt] \textrm{II} & 4 - 16a & = & -8 \\ \end{array} \)

\(\\\)

Wir berechnen \(a\) mit Gleichung \(\textrm{I}\).

\( \quad \begin{array}{ r c c l } 4a & = & 3 & | : 4 \\[6pt] a & = & \frac{3}{4} \\ \end{array} \)

\(\\\)

Wir machen die Probe mit \(\textrm{II}\).

\( \quad \begin{array}{ r c c l } 4 - 16 \cdot \frac{3}{4} & = & - 8 \\[6pt] -8 & = & - 8 \\ \end{array} \)

\(\\\)

Damit ist \(a = \frac{3}{4}\).

\(\\[2em]\)

Aufgabe 2 Schnittwinkel

\( \quad \begin{array}{ r c c l } E_8 \; : & 4x_1+4x_2 - 8x_3 & = & 1 - 4 \cdot 8 \\[6pt] & 4x_1+4x_2 - 8x_3 & = & - 31 \\ \end{array} \)

\(\\\)

Den Schnittwinkel der beiden Ebenen berechnen wir mit

\( \quad cos(\alpha) = \dfrac{ \begin{vmatrix} \vec{n_8} \circ \vec{n_{\frac{3}{4}}} \end{vmatrix} } { \begin{vmatrix} \vec{n_8} \end{vmatrix} \cdot \begin{vmatrix} \vec{n_{\frac{3}{4}}} \end{vmatrix} } \)

\(\\\)

Wir verwenden dabei

\( \quad \vec{n_8}= \begin{smallmatrix} \left( \begin{array}{r} 4 \\ 4 \\ -8 \end{array} \right) \end{smallmatrix} \)

\(\\\)

und

\( \quad \vec{n_{\frac{3}{4}}} = \begin{smallmatrix} \left( \begin{array}{r} 4 \\ 4 \\ -3 \end{array} \right) \end{smallmatrix} \)

\(\\\)

Es gilt

\( \quad \begin{array}{ r c l } cos(\alpha) & = & \frac{ \begin{vmatrix} \begin{smallmatrix} \left( \begin{array}{r} 4 \\ 4 \\ -8 \end{array} \right) \end{smallmatrix} \circ \begin{smallmatrix} \left( \begin{array}{r} 4 \\ 4 \\ -3 \end{array} \right) \end{smallmatrix} \end{vmatrix} } { \begin{vmatrix} \begin{smallmatrix} \left( \begin{array}{r} 4 \\ 4 \\ -8 \end{array} \right) \end{smallmatrix} \end{vmatrix} \cdot \begin{vmatrix} \begin{smallmatrix} \left( \begin{array}{r} 4 \\ 4 \\ -3 \end{array} \right) \end{smallmatrix} \end{vmatrix} } \\[12pt] cos(\alpha) & = & \frac{\bigl| 4 \cdot 4 + 4 \cdot 4 - 8 \cdot (-3) \bigl| } { \sqrt{ 4^2 +4^2 + (-8)^2 } \cdot \sqrt{ 4^2 +4^2 + (-3)^2 } } \\[10pt] cos(\alpha) & = & \frac{ 56 } { \sqrt{ 96 } \cdot \sqrt{ 41 } } \\[12pt] \alpha & = & cos^{-1}\left( \frac{56}{\sqrt{96} \cdot \sqrt{41}} \right) \; = \; 26{,}8^\circ \\ \end{array} \)

\(\\[2em]\)

Aufgabe 3 Winkel von 60°

Wir verwenden wieder die Winkelformel, dieses Mal mit vorgegebenen Winkel.

\( \quad cos(60^\circ) = \dfrac{ \begin{vmatrix} \vec{n_a} \circ \vec{n_{\frac{3}{4}}} \end{vmatrix} } { \begin{vmatrix} \vec{n_a} \end{vmatrix} \cdot \begin{vmatrix} \vec{n_{\frac{3}{4}}} \end{vmatrix} } \)

\(\\\)

Dabei ist

\( \quad \vec{n_a}= \begin{smallmatrix} \left( \begin{array}{r} 1 \\ 1 \\ -a \end{array} \right) \end{smallmatrix} \)

\(\\\)

Es gilt

\( \quad \begin{array}{ r c l l } cos(60^\circ) & = & \frac{ \begin{vmatrix} \begin{smallmatrix} \left( \begin{array}{r} 1 \\ 1 \\ -a \end{array} \right) \end{smallmatrix} \circ \begin{smallmatrix} \left( \begin{array}{r} 4 \\ 4 \\ -3 \end{array} \right) \end{smallmatrix} \end{vmatrix} } { \begin{vmatrix} \begin{smallmatrix} \left( \begin{array}{r} 1 \\ 1 \\ -a \end{array} \right) \end{smallmatrix} \end{vmatrix} \cdot \begin{vmatrix} \begin{smallmatrix} \left( \begin{array}{r} 4 \\ 4 \\ -3 \end{array} \right) \end{smallmatrix} \end{vmatrix} } \\[12pt] cos(60^\circ) & = & \frac{ \begin{vmatrix} 1 \cdot 4 + 1 \cdot 4 - a \cdot (-3) \end{vmatrix} } { \sqrt{ 1^2 +1^2 + (-a)^2 } \cdot \sqrt{ 4^2 +4^2 + (-3)^2 } } \\[10pt] 0{,}5 & = & \frac{\begin{vmatrix} 8 + 3a \end{vmatrix} } { \sqrt{2 + a^2} \cdot \sqrt{ 41 } } & \bigl| \; ( \dots)^2 \\[10pt] 0{,}5^2 & = & \left( \frac{\begin{vmatrix} 8 + 3a \end{vmatrix} } { \sqrt{2 + a^2} \cdot \sqrt{41} } \right)^2 \\ \end{array} \)

\(\\\)

Die Klammer lösen wir mit dem 2.Potenzgesetz auf.

\( \quad \begin{array}{ r c l l } 0{,}5 & = & \frac{\begin{vmatrix} 8 + 3a \end{vmatrix}^2 } { \sqrt{2 + a^2}^2 \cdot \sqrt{41}^2 } \\[10pt] 0{,}5 & = & \frac{( 8 + 3a )^2}{ \left( 2 + a^2 \right) \cdot 41 } & | \cdot 41 \left( 2 + a^2 \right) \\[10pt] 41 + 20{,}5a^2 & = & 64 + 48a + 9a^2 & | - 41 \\[8pt] 20{,}5a^2 & = & 9a^2 + 48a + 23 & | - 20{,}5a^2 \\[8pt] 0 & = & -11{,}5a^2 + 48a + 23 \\ \end{array} \)

\(\\\)

Die Gleichung kann mit der PQ-Formel oder wie hier gezeigt, mit der Nullstellenberechnung von Polynomen gelöst werden.

Nach \(\boxed{MENU}\) gehen wir mit Pfeil rechts nach

my image

\(\\\)

und wählen dies mit \(\boxed{=}\) aus.

my image

\(\\\)

Wir wählen \(\boxed{2}\)

my image

\(\\\)

und noch einmal \(\boxed{2}\).

my image

\(\\\)

Wir geben die Parameter ein

my image

\(\\\)

und bestätigen mit \(\boxed{=}\). Wir erhalten

my image

\(\\\)

Mit \(\boxed{S \Leftrightarrow D}\) wird der Wert dezimal angezeigt.

my image

\(\\\)

Wir bestätigen mit \(\boxed{=}\) und lassen uns entsprechend den 2. Wert anzeigen.

my image

my image

\(\\\)

Die möglichen \(a\)-Werte lauten also

\( \quad a_1=4{,}607945916 \quad \text{ und} \quad a_2=-0{,}4340328721 \)

\(\\\)